5
$\begingroup$

I am interested in determining the behavior of the following double integral $$ I_N = \int\limits_{0}^{1} \int\limits_{0}^{2 \pi} \Big[ \big( (2x-1)(1+\cos t) -i \sin t \big) x (1-x) \Big]^{N} \, dt \, dx $$ as $N \to \infty$ (one can also write the part depending on $t$ in terms of $e^{\pm it}$ if it helps).

From numerics the integral appears to be exponentially small in $N$ (like $\approx e^{-1.6 N}$, probably up to polynomial factors). However, when one writes the integrand as $f(x,t)^N = e^{N g(x,t)}$, the function $g(x,t)$ has both real and imaginary part, so asymptotic methods such as Laplace's method or stationary phase as I know them are not available out of the box. One source that I found which treats the multidimensional case in some generality is Pemantle and Wilson's paper "Asymptotic expansions of oscillatory integrals with complex phase". In any case, the function $g$ does not have any critical points inside the region of interest, so I am at a loss as how to proceed.

When integrated over $t$, the integrand has its peak at $x = 1/2$, but there are multiple smaller peaks which cancel out each other to some extent, overall it looks like the cancellations are somewhat subtle. Any help would be appreciated.

$\endgroup$
1
  • 3
    $\begingroup$ Note that $(2x-1) \in [-1,1]$ in your domain, $(1+\cos t)\in [0,2]$ and $\sin t \in [-1,1]$. Your integrand is therefore bounded by $$ | \sqrt{5} x(1-x) |^N $$ Now, the function $x(1-x)$ attains its maximum at $x = 1/2$ with the value $1/4$. So you get that the integrand is bounded by $(5/16)^{N/2} \approx \exp(-0.6N)$. You won't get any help from oscillatory decay. $\endgroup$ Commented Apr 16 at 23:22

2 Answers 2

4
$\begingroup$

The double integral has a closed form of zero for odd indices and for even index

$I_{2n}= (-1)^n \dfrac{2^{2n+1}}{(n-1/6)!} \dfrac{3^{-3n-1}}{(n+1/6)!} \big( (n-1/2)! \big)^2 \, \pi$

With Gamma function expansion, $I_{2n} \sim (-1)^n \dfrac{2\pi}{3n}\exp{(-3n\log{3})} \big(1-\dfrac{5}{18n} + ...\big) $

Proof sketch follows: Write, with binomial theorem,

$I_n=\int_0^1 (x(1-x))^n \sum_{k=0}^n \binom{n}{k}( 2(2x-1))^k(-2i)^{n-k} \int_0^{2\pi}\cos^{n+k}(t/2) \sin^{n-k}(t/2) dt$

By Beta integral in trig form, the $dt$ integral is solvable in closed form:

$I_n=\int_0^1 (x(1-x))^n \sum_{k=0}^n \binom{n}{k}( 2(2x-1))^k(-2i)^{n-k} \frac{1}{2}(1+(-1)^{n-k})\Gamma(\frac{n+k+1}{2})\Gamma(\frac{n-k+1}{2})/n! $

For $dx$ integral, write $(2x-1)^k = (x-(1-x))^k$ and again use binomial theorem and Beta integral to get a closed form. Wonderfully we find that

$\sum_{m=0}^k \binom{k}{m}(-1)^{k-m}\int_0^1 x^{n+m} (1-x)^{n+k-m} dx = 4^{-1-n}(1+(-1)^k)\Gamma(\frac{k+1}{2})n!/\Gamma(\frac{k+3}{2} + n)$

where the closed-form was generated by Mathematica. Note now that we have factors $(1+(-1)^k) (1+(-1)^{n-k}) $ which implies that both $n$ and $k$ must be even. Collect results thus far...

$I_{2n}=4^{-n}\sum_{k=0}^n\binom{2n}{2k}(-1)^{n-k}\frac{\Gamma(k+1/2)}{\Gamma(2n+k+3/2)} \Gamma(n+k+1/2)\Gamma(n-k+1/2). $

Mathematica can solve in closed-form this sum, which appears as the answer. Readers, please feel free to edit for clarity and a prettier format; too much time went into finding the solution, and not enough for presentation.

$\endgroup$
2
  • $\begingroup$ This is amazing - actually I found my integral by considering a certain alternating combinatorial sum with binomial coefficients etc., which, alas, Mathematica could not simplify to an exact answer. That's why I was looking for asymptotics. $\endgroup$ Commented Apr 22 at 9:21
  • $\begingroup$ Glad to help. It's rare that there's a question on Overflow in my wheelhouse, and I haven't looked on this site for a few years. $\endgroup$ Commented Apr 23 at 3:04
2
$\begingroup$

$\newcommand\ep\varepsilon$Write $$I_n=\int_0^1 dx\int_{-\pi}^\pi dt\, f(t,x)^n,$$ where $f(t,x):=((2x-1)(1+\cos t)-i\sin t)x(1-x)$. The maximum value $1/4$ of $|f(t,x)|$ over all $(t,x)\in[-\pi,\pi]\times[0,1]$ is attained only at the points $(\pm\pi/2,1/2)$. So, it is easy to see that $$I_n=(-i/4+o(1))^n=(-1)^m(1/4+o(1))^{2m} \tag{1}\label{1} $$ for large even $n=2m$. Actually, for even $n$ the value of $I_n$ is real, because $f(-t,x)$ is the complex conjugate of $f(t,x)$, and the bijection $(t,x)\mapsto(-t,x)$ of the set $[-\pi,\pi]\times[0,1]$ preserves the Lebesgue measure.

For odd $n$, we have $I_n=0$, because $f(-t,1-x)=-f(t,x)$ for $(t,x)\in[-\pi,\pi]\times[0,1]$, and the bijection $(t,x)\mapsto(-t,1-x)$ of the set $[-\pi,\pi]\times[0,1]$ preserves the Lebesgue measure.


For an illustration of \eqref{1}, here is the graph $\big\{\big(m,((-1)^m I_{2m})^{1/(2m)}\big)\colon m=1,\dots,100\big\}$ (blue), with a dashed horizontal line at level $1/4$:

enter image description here

$\endgroup$
5
  • $\begingroup$ How do you obtain more information about the $o(1)$ term? Unfortunately, for me $(1/4 + o(1))^{2m}$ is not enough, I'd rather have $h(m)(1/4)^{2m}(1 + o(1))$ for some $h$, possibly with some bounds on the $o(1)$ remainder. $\endgroup$ Commented Apr 17 at 9:15
  • 1
    $\begingroup$ @MichałKotowski The integrand is approximately gaussian near the two critical points and this can be used to get an accurate value. $\endgroup$ Commented Apr 17 at 14:38
  • 1
    $\begingroup$ @MichałKotowski : I underestimated the complexity caused by the imaginary part. Will try to clarify this. $\endgroup$ Commented Apr 17 at 17:46
  • $\begingroup$ This is indeed what confuses me, since the complex phase itself is not stationary, even though the modulus is. $\endgroup$ Commented Apr 19 at 9:28
  • 2
    $\begingroup$ As a matter of fact, it seems to me that the contribution of $(x,t) = (1/2, \pm \pi / 2)$ will not be $\sim (1/4)^n$, but smaller (like $\sim (1/4 \cdot e^{-1/4})^n$, if I made the calculations correctly). If one does Gaussian approximation for $f(x,t) = e^{g(x,t)}$ like Brendan McKay suggested, then the point $(1/2, \pm \pi / 2)$ is not critical for $g$ (it is only critical for the modulus), so after change of variables there will be a correction term (which is why I have $e^{-1/4}$). Then it seems it is no longer straightforward to argue that the main contribution comes from this point only. $\endgroup$ Commented Apr 21 at 18:04

You must log in to answer this question.

Start asking to get answers

Find the answer to your question by asking.

Ask question

Explore related questions

See similar questions with these tags.